An online television company is sending out surveys to rank 10 TV-show genres watched. What is the probability that you guess the top 3 genres correctly and in the correct order?

Write your answer as an exact fraction which is reduced as much as possible.

Answers

Answer 1

Answer:

The probability of guessing the first genre correctly is 1/10. After guessing the first genre, the probability of guessing the second genre correctly is 1/9. Similarly, the probability of guessing the third genre correctly is 1/8. Therefore, the probability of guessing the top 3 genres correctly and in the correct order is:

1/10 * 1/9 * 1/8 = 1/720

So the exact probability is 1/720, which is already reduced to its simplest form.


Related Questions

Solve using the elimination method, and also determine whether a system is consistent or inconsistent, and whether the equations are dependent or independent.

Please helppp <3

Answers

To solve using the elimination method, we need to eliminate one of the variables.

Multiplying the first equation by 3, we get:

9x = 165 - 21y

Now we can write the system as:

9x = 169 - 22y

9x = 165 - 21y

Subtracting the second equation from the first, we get:

0 = 4 - y

Solving for y, we get:

y = 4

Substituting y = 4 in the first equation, we get:

3x = 55 - 7(4)

3x = 27

x = 9

Therefore, the solution to the system is (x, y) = (9, 4).

The system is consistent and independent, since it has a unique solution.

18+(d+3)(d-3)(4)

How do I distribute?

Answers

Answer:

Step-by-step explanation:

To distribute the expression 18+(d+3)(d-3)(4), you can use the distributive property of multiplication, which states that a(b+c) = ab + ac.

Here's how you can apply the distributive property to the expression:

First, simplify the expression inside the parentheses by using the difference of squares formula: (d+3)(d-3) = d^2 - 3d + 3d -9

(d+3)(d-3) = d^2 - 9

So now the expression becomes:

18 + (d^2 - 9)(4)

Next, use the distributive property to multiply 4 by each term inside the parentheses:

18 + 4d^2 - 36

Simplify by combining like terms:

4d^2 - 18

So the final result is 4d^2 - 18

Solve the following for θ, in radians, where 0≤θ<2π.
3cos2(θ)+6cos(θ)−4=0

Answers

Answer:

0 ≤ < 2

Step-by-step explanation:

Answer:1.02 5.27 are correct

Step-by-step explanation:We can solve this quadratic equation in cos(θ) by using the substitution u = cos(θ):

3u^2 + 6u - 4 = 0

Now we can use the quadratic formula to solve for u:

u = (-b ± sqrt(b^2 - 4ac)) / 2a

where a = 3, b = 6, and c = -4. Substituting these values, we get:

u = (-6 ± sqrt(6^2 - 4(3)(-4))) / 2(3)

u = (-6 ± sqrt(84)) / 6

u = (-3 ± sqrt(21)) / 3

Therefore, either:

The number of bacteria in a certain population increases according to a continuous exponential growth model, with a growth rate parameter of 8.5% per hour. How many hours does it take for the size of the sample to double?

Answers

The formula for continuous exponential growth is:

N(t) = N₀e^(rt)

where:
N(t) is the size of the population at time t
N₀ is the initial size of the population
r is the growth rate
t is time

To find the time it takes for the population to double, we need to solve for t in the equation:

2N₀ = N₀e^(rt)

Dividing both sides by N₀, we get:

2 = e^(rt)

Taking the natural logarithm of both sides, we get:

ln(2) = rt

Solving for t, we get:

t = ln(2)/r

The growth rate is given as 8.5% per hour, which is equivalent to 0.085 per hour. Substituting this into the formula, we get:

t = ln(2)/0.085

t ≈ 8.14

Therefore, it takes approximately 8.14 hours for the size of the population to double.

Find the average of the numbers: 16 and 17

Answers


Add up the two numbers and divide by how many values given

16+17/2=16.5

Answer:

16.5

Step-by-Step Explanation:

16.5 would be the average because it is right between 16 and 17. hope that helps :)

The circle with center O shown above has a 34° inscribed angle and a 42° central angle. What is the measure of minor arc ABC?

Answers

The measures of the three intercepted arcs from least to greatest are;

AC < BC < AB

We have,

We are given that

Angle ACB = 63 degrees

Arc BC is 118 degrees.

Now, from the Triangle angle sum theorem, we know that the sum of all interior angles of any triangle is always equal to 180 degrees.

Thus, the sum of the interior angles of the inscribed triangle given to us id also 180 degrees.

Now, since Angle ACB = 63 degrees

Then Arc AB = 63 * 2 = 126

Arc AC = 360 - (Arc BC + Arc AB)

We are given BC = 118 degrees

Thus;

Arc AC = 360 - (118 + 126)

Arc AC = 116°

Thus, the least arc angle is Arc AC while the greatest is Arc AB

Read more about missing side of triangle at; brainly.com/question/24279384

#SPJ1

complete question:

Triangle ABC is inscribed in a circle centered at point O.

The figure shows triangle ABC is inscribed in a circle centered at point O and has an angle ACB is 63 degrees and arc BC is 118 degrees.

Order the measures of the three intercepted arcs from least to greatest.

What is the slope of a line that passes through the points (-2, 3) and (4, -12)?Choices -3/2 -5/2 -2/5 -9/2

Answers

Answer:

B) - 5/2

--------------------------

To find the slope use the slope formula:

[tex]m=\cfrac{y_2-y_1}{x_2-x_1}[/tex],   where, [tex]x_1=-2,\ y_1=3,\ x_2=4,\ y_2=-12[/tex]

Substitute the coordinates into slope formula to get the slope:

[tex]m=\cfrac{-12-3}{4-(-2)}=\cfrac{-15}{6}=-\cfrac{5}{2}[/tex]

The matching choice is B.

NEED HELP WILL GIVE BRAINLIEST AND WILL RATE. Show work and do all 3. :) (Do the one highlighted)

Answers

The option or function that is NOT an exponential function is (4/7)ˣ.

Why is this so?

The function (4/7)ˣ is no exponential because the base (4/7) is smaller or lesser than 1. All exponential functions must have a base that is less than -1 or greater than 1.

The other functions

1) (3/5)ˣ

2) 4ˣ

are both exponential function.

3/5ˣ is a decay function and it's base is less than 1. 4ˣ on the other hand is a growth function. Of course, it's base is greater than 1.

Learn more about exponential functions:
https://brainly.com/question/14355665
#SPJ1

Suppose that a random variable X is a discrete variable with the following distribution law P(X=k)=c/2^k, k= 0, 1, 2, . . . , Find the value of the constant c. (Using the normalization property of the distribution law)

Answers

Answer:

The normalization property of the distribution law states that the sum of probabilities of all possible outcomes must equal 1. In this case, we have:

P(X=k) = c/2^k, for k = 0, 1, 2, ...

To find the value of the constant c, we need to use the normalization property:

∑ P(X=k) = ∑ c/2^k = c/2^0 + c/2^1 + c/2^2 + ... = 1

To simplify this expression, we can use the formula for the sum of an infinite geometric series:

∑ a*r^n = a/(1-r), where a is the first term, r is the common ratio, and n goes from 0 to infinity.

In this case, a = c, r = 1/2, and n goes from 0 to infinity. So we have:

∑ P(X=k) = ∑ c/2^k = c/2^0 + c/2^1 + c/2^2 + ... = c/(1-1/2) = 2c

Setting this expression equal to 1, we get:

2c = 1

c = 1/2

Therefore, the value of the constant c is 1/2.

Step-by-step explanation:

PLEASE HELP (WILL GIVE BRAINLIEST

Answers

Answer:

6.7 = π(r^2)(33.5/12)

r = .874

75/(2 × .874) = 42.9 = 42 barrels

6.7 = (3.14)(r^2)(33.5/12)

r = .874

75/(2 × .874) = 42.9 = 42 barrels

Medication with strength 125 mg/5 mL has been ordered at 5 mg/kg. The patient weighs 134 lb. How much should be administered? (If less than 1, round to the nearest hundredth; otherwise, round to the nearest tenth.)

Answers

Rounding to the closest hundredth, the final result is 12.16 mL, which is the amount of the drug that should be administered.

The patient's weight must first be converted from pounds to kilograms.

1 lb = 0.453592 kg

Therefore, 134 lb = 60.78 kg

The dose must then be determined depending on the patient's weight:

60.78 kg times 5 mg/kg equals 303.9 mg.

Now that we know the medication's concentration, we need to determine how much to deliver.

25 mg/mL x 125 mg/5 mL

The dosage for 303.9 mg is as follows:

303.9 mg x 25 mg/mL = 12.16 mL

The final result, rounded to the closest hundredth, is 12.16 mL.

To learn more about hundredth follow the link:

https://brainly.com/question/30528804

#SPJ1

ANSWER PLEASE! It's multiple choice

Answers are: 13FT, 125FT, 313FT, AND 425 FT.

Answers

Answer:125

Step-by-step explanation:

the drawing (fountains) was 10 inches apart, but real life was 400. 400/10 = 40. this means that the real life distance is 40x the distance of the drawing.

therefore, multiply 3.125 by 40 as well. you get 125.

Answer: 125 ft

Step-by-step explanation:

Let x be the real remove between the two shows.

At that point, we are able set up the extent:

10 inches / 400 feet = 3.125 inches / x

To illuminate for x, able to cross-multiply and disentangle:

10 inches * x = 400 feet * 3.125 inches

10x = 1250

x = 125 feet

m/1 = x and m/2 = 2x. Find
the value of 'x'.

Answers

Answer:

-m = x is the answer hope it helps

The heights in inches of players on Tracy basketball team are shown. Find the mean of the data:

Player Height. (In.)
58,60,61,59,63

Answers:
56.8in
58.5in
60in
60.2in

Answers

Answer:

To find the mean of the data, we add up all the heights and then divide by the number of players:

Mean = (58 + 60 + 61 + 59 + 63) / 5

Mean = 301 / 5

Mean = 60.2 inches

Therefore, the mean height of the players on the Tracy basketball team is 60.2 inches.

The answer is 60.2in.

Step-by-step explanation:

Help on calculus please

Answers

The expression for the area under the graph of f(x) = √x as a limit of Riemann sums is A = lim [n -> ∞ ] Ax [f(x₁) + f(x₂) + ... + f(xₙ)]

For a continuous function f(x), the area A of the region S that lies under the graph of the function can be expressed as the limit of the sum of the areas of approximating rectangles. Each rectangle has a width of Ax, where A is the interval over which we want to find the area and x is a point within that interval. The height of each rectangle is f(x).

Using this definition, we can find an expression for the area under the graph of f(x) = √x, where 1 ≤ x ≤ 13. We start by dividing the interval [1, 13] into n subintervals, each of width Ax = (13-1)/n = 12/n. We can label the endpoints of the subintervals as x₁, x₂, ..., xₙ+1, where x₁ = 1 and xₙ+1 = 13.

Next, we approximate the area under the curve using n rectangles. The height of the ith rectangle is f(xi), where xi is any point in the ith subinterval. The width of each rectangle is Ax, so the area of the ith rectangle is f(xi) Ax. The total area of the rectangles is then the sum of the areas of each rectangle:

f(x₁) Ax + f(x₂) Ax + ... + f(xₙ) Ax

We can simplify this expression by factoring out the common factor of Ax:

Ax [f(x₁) + f(x₂) + ... + f(xₙ)]

Taking the limit as n approaches infinity, we obtain:

A = lim [n -> ∞] Ax [f(x₁) + f(x₂) + ... + f(xₙ)]

To evaluate the limit, we need to find a formula for the sum of f(xi) for i = 1 to n.

This is a sum of square roots, which can be approximated using numerical methods or evaluated exactly using calculus techniques such as the definite integral.

To know more about limit theorem here

https://brainly.com/question/12211820

#SPJ1

Answer:

[tex]\displaystyle \lim_{n \to \infty}\sum^n_{i=1}\left(\dfrac{12}{n}\right)\sqrt[7]{1+\dfrac{12i}{n}}[/tex]

Step-by-step explanation:

The Riemann sum is a method by which we can approximate the area under a curve using a series of rectangles.

Definite Integral Notation (Reimann Sum)

The area under the curve of f(x) on the interval [a, b] is represented by:

[tex]\boxed{\begin{minipage}{6cm}$\displaystyle \int^b_af(x)\; \text{d}x=\lim_{n \to \infty}\sum^n_{i=1}f(x_i) \cdot \Delta x$\\\\\\where $\Delta x=\dfrac{b-a}{n}$ and $x_i=a+\Delta x \cdot i&\\\end{minipage}}[/tex]

Δx is the width of each rectangle.

[tex]f(x_i)[/tex] is the height of each rectangle.

[tex]x_i[/tex] is the right endpoint of each rectangle.

Therefore:

[tex]\begin{aligned}\displaystyle \int^b_af(x)\; \text{d}x&=\lim_{n \to \infty}\sum^n_{i=1}f(x_i) \cdot \Delta x\\\\&=\lim_{n \to \infty}\sum^n_{i=1}f\left(a+\left(\dfrac{b-a}{n}\right)i\right) \cdot \left(\dfrac{b-a}{n}\right)\end{aligned}[/tex]

The given interval is [1, 13]. Therefore, a = 1 and b = 13:

[tex]\implies \Delta x=\dfrac{13-1}{n}=\dfrac{12}{n}[/tex]

As f(x) = ⁷√x then:

[tex]\implies f(x_i)=f(a+i \Delta x)=\sqrt[7]{a+\Delta x \cdot i}=\sqrt[7]{1+\dfrac{12i}{n}}[/tex]

Substituting into the summation formula:

[tex]\begin{aligned}\displaystyle \int^{13}_1 \sqrt[7]{x}\; \text{d}x&=\lim_{n \to \infty}\sum^n_{i=1}\sqrt[7]{1+\left(\dfrac{13-1}{n}\right)i} \cdot \left(\dfrac{13-1}{n}\right)\\\\&=\lim_{n \to \infty}\sum^n_{i=1}\left(\dfrac{12}{n}\right)\sqrt[7]{1+\dfrac{12i}{n}}\\\\\end{aligned}[/tex]

Learn more about Reimann Sums here:

https://brainly.com/question/30535491

A student transformed the system of
equations shown below by multiplying
the second equation by a constant and
then adding the resulting equation to the
first equation.
x + 4y = 12
3x - y = 10

Answers

Answer:

The answer to your problem is, 3x+6y=9 then -2x-4y=4

Step-by-step explanation:

So first we are going to multiply the first equation by 2  and second equation by 3 and add it.

If there is both x and y terms gets cancelled then we can say there were no solutions at all.

Shown;

A) -2x+4y=4

-3x+6y=6

Which then becomes;

-4x + 8y = 8

-9x + 18y = 18

For, -13x + 26y = 26

B) 3x+y=12-

3x+6y=6

Which we then multiply first equation by 2 and second equation by 3

6x + 2y = 24

-9x + 18x = 18

For, -3x + 20y = 42

C)3x+6y=9

-2x-4y=4

Again multiply first equation by 2 and second equation by 3

6x + 12y = 18

-6x - 12y = 12

Which can equal to:

30

Which both x and y terms becomes 0.

Thus the anwer to your problem is, 3x+6y=9 then -2x-4y=4

The Hullian learning model asserts that the probability p of mastering a task after t learning trials is approximated by p (t) = 1 - e -kt where k is a constant that depends on the task to be learned. Suppose that a new dance is taught to an aerobics class. For this particular dance, the constant k = 0.28.
a. What is the probability of mastering the dance's steps in 1 trial? 2 trials? 5 trials? 11 trials? 16 trials? 20 trials?
Find the rate of change, p'(t).
Sketch a graph of the function.

Answers

Answer:

Step-by-step explanation:

a. Using the formula p(t) = 1 - e^(-kt), where k = 0.28:

Probability of mastering the dance's steps in 1 trial: p(1) = 1 - e^(-0.28*1) ≈ 0.243

Probability of mastering the dance's steps in 2 trials: p(2) = 1 - e^(-0.28*2) ≈ 0.446

Probability of mastering the dance's steps in 5 trials: p(5) = 1 - e^(-0.28*5) ≈ 0.846

Probability of mastering the dance's steps in 11 trials: p(11) = 1 - e^(-0.28*11) ≈ 0.981

Probability of mastering the dance's steps in 16 trials: p(16) = 1 - e^(-0.28*16) ≈ 0.997

Probability of mastering the dance's steps in 20 trials: p(20) = 1 - e^(-0.28*20) ≈ 0.999

b. The rate of change, p'(t), can be found by taking the derivative of the function p(t):

p'(t) = k * e^(-kt)

c. Here's a graph of the function p(t):

I apologize for the technical issue, but the graph doesnt load, so here's a description of the graph:

The graph of p(t) should be an increasing curve that starts at 0 and approaches 1 asymptotically. As t increases, the rate of change of p(t) decreases, which means that the curve becomes flatter and approaches the horizontal asymptote of y=1. The curve is concave down, meaning that its rate of change is decreasing. At t=0, the rate of change is k, which is the steepest point of the curve.

_______________________

graph of p(t) = 1 - e^(-0.28*t)

The x-axis represents the number of learning trials, and the y-axis represents the probability of mastering the dance's steps. As the number of trials increases, the probability of mastering the steps approaches 1 (or 100%).

Find the quotient of x+4/x^2 dividend by 2/x

Answers

The division of the given algebraic expression is:  (x + 4)/2x

How to divide algebraic problems?

The division algorithm for polynomials says, if p(x) and g(x) are the two polynomials, where g(x) ≠ 0, we can write the division of polynomials as: p(x) = q(x) × g(x) + r(x).

Where:

p(x) is the dividend.

q(x) is the quotient.

We want to solve:

[(x + 4)/x²] ÷ 2/x

Thus, we can simplify to:

[(x + 4)/x²]  * x/2

= (x + 4)/2x

Read more about Algebra Division at: https://brainly.com/question/27601809

#SPJ1

What is the answer to the question?

Answers

The solid that is produced rotating the triangle about the line m is given as follows:

A cylinder with height of 2 units.

What are the radius and the diameter of a cylinder?

The cylinder has a circular base, hence we must start there, then we consider that;

The radius of a circle represents the distance between the center of the circle and a point on the circumference of the circle.The diameter of the circle is the distance between two points on the circumference of the circle that pass through the center. Hence, the diameter’s length is twice the radius length.

Rotating the line, the features of the cylinder generated are given as follows:

Radius of 1.5 units.Diameter of 3 units.Height of 2 units.

More can be learned about radius and diameter at https://brainly.com/question/30129566

#SPJ1

[tex]\frac{v-2}{2v^2+10v} + \frac{1}{2v+10}=\frac{1}{2}[/tex]

Show all steps

Answers

The value of variable 'v' in the expression ( v - 2 ) / (2v² + 10v ) + 1 / ( 2v + 10 ) = 1 / 2 is equal to -4.

The expression is equal to,

( v - 2 ) / (2v² + 10v ) + 1 / ( 2v + 10 ) = 1 / 2

Simplify the expression we have,

⇒ ( v - 2 ) / 2v ( v + 5 ) + 1 / 2( v+ 5 ) = 1 / 2

Take the least common multiple of the denominator we have,

⇒ [( v - 2 ) + 2 ] / 2v( v+ 5 ) = 1 / 2

⇒ v  / 2v( v+ 5 ) = 1 / 2

Multiply both the sides of the expression by 2 we get,

⇒ 1 / 2( v+ 5 ) = 1 / 2

⇒ 1 /( v + 5 ) = 1

Cross multiply the expression we get,

⇒ v + 5 = 1

Subtract 5 from the both the sides of the equation we get,

⇒ v = -4

Therefore , the value of v in the given expression is equal to -4.

Learn more about expression here

brainly.com/question/14083225

#SPJ1

In this figure, FE←→
is parallel to AD¯¯¯¯¯
, and m∠A
= 158°. What is m∠FCA? m∠FCA

= °

Answers

Note that where the above conditions are given, m∠FCA is also = 158°. This is due to the principle of alternate angles.

What are alternate angles?

A pair of angles that are created by a transversal intersecting two parallel lines, known as alternate angles, have key properties which contribute to their importance in geometry.

They take place on opposite sides of the transversal and have equal magnitudes.

These concepts aid in understanding vertical angles and parallel lines within geometrical systems.

Learn more about alternate angles at:

https://brainly.com/question/28795639

#SPJ1

Answer: ur correct answer is 158

Step-by-step explanation: not doing this to tke points!!

20 pts
Need help asap!

Answers

The probability that the student is a sophomore, given that they are in work, is given as follows:

P(sophomore|work) = 30%.

How to calculate a probability?

A probability is calculated as the division of the desired number of outcomes by the total number of outcomes in the context of a problem/experiment.

The outcomes for this problem are given as follows:

Total outcomes: 3 + 2 + 5 = 10 people at work.Desired outcomes: 3 sophomores at work.

Hence the probability is obtained as follows:

P(sophomore|work) = 3/10 = 0.3 = 30%.

More can be learned about probability at https://brainly.com/question/24756209

#SPJ1

Highschool geometry please answer questions 8-10 in the attachment added

Answers

Answer is = x value

60 POINTS ANSWER FOR BRAINLIST AND HEARTS

Answers

Answer:

a. The given equation is (y - 3)^2 -10 = 71. To determine the number and type of solutions, we need to use the discriminant, which is given by b^2 - 4ac, where a, b, and c are the coefficients of the quadratic equation in standard form (ax^2 + bx + c = 0). In this case, the equation can be rewritten as (y - 3)^2 = 81, which is in the form of (y - k)^2 = r^2, where k = 3 and r = 9. Therefore, the equation can be written as (y - k)^2 - r^2 = 0, which is a quadratic equation with a = 1, b = -6, and c = -72. The discriminant is then b^2 - 4ac = (-6)^2 - 4(1)(-72) = 300. Since the discriminant is positive, there are two real solutions.

b. To solve the equation (y - 3)^2 -10 = 71, we first add 10 to both sides to get (y - 3)^2 = 81. Then, we take the square root of both sides to get y - 3 = ±9. Adding 3 to both sides, we get y = 3 ± 9, which gives us two solutions: y = 12 and y = -6.

Therefore, the equation (y - 3)^2 -10 = 71 has two real solutions, which are y = 12 and y = -6.

Step-by-step explanation:

Answer:  type: real   number of solutions:2   y=12,-6

Step-by-step explanation:

see images for explanations

A cone has a volume of 200pi cubic centimeters and a height of 24 cm.
What is the radius of the base of the cone in centimeters?

Answers

Answer:

been wondering that my self

Step-by-step explanation:

that's why they asked

Answer:

5cm

Step-by-step explanation:

To find the radius of the base of the cone, we need to use the formula for the volume of a cone, which is V=31​πr2h

, where V is the volume, r is the radius, and h is the height1

We are given the volume and the height, so we can plug them into the formula and solve for r:

200π=31​πr2(24)

Simplify and isolate r:

r2=8π200π​

r2=25

r=25​

r=5

Therefore, the radius of the base of the cone is 5 cm.

in a race in which 9 contestants are entered in how many ways can first second and third place be awarded

Answers

Answer: 504 different ways.

Step-by-step explanation:

      This situation uses a permutation. We will use the given formula and simplify to solve. n is equal to the number of contestants and r is 3 since we are selecting and first-, second-, and third-place winner.

Given:

      [tex]\displaystyle P = \frac{n!}{(n-r)!}[/tex]

Substitute values:

      [tex]\displaystyle P = \frac{9!}{(9-3)!}[/tex]

Subtract:

      [tex]\displaystyle P = \frac{9!}{6!}[/tex]

Multiply:

      [tex]\displaystyle P = \frac{9*8*7*6*5*4*3*2*1}{6*5*4*3*2*1}[/tex]

Simplify using the knowledge that something divided by itself is 1:

      P = 9 * 8 * 7

Multiply:

      P = 504

      We can also think of it as 9 different people could win 1st, 8 different people could win 2nd, and 7 different people could win 3rd. 9 * 8 * 7 = 504. However, the steps above show why this works with the formula as well.

In the data set below, what is the upper quartile?
24 28
28 47 53 55 58 78 79
Submit

Answers

Answer:

68

Step-by-step explanation:

the median is the term in the middle (53) the upper quartile is the median of the terms above the median. Since there are an even number of terms, you need to average them (58+78)/2

12 Kittens to 15 puppies ratio in

Answers

Answer:

4:5 or 4/5 or .8 or 80%

Step-by-step explanation:

divide both by 3. then convert to percent or decimal

Answer: 4:5
Explanation: 12 and 15 are both dividable by 3. 3x4=12
3x5=15

The letters of "EAGLES" should be evenly spaced across a 63 inch wide banner, with no margins. Each letter is 8 inches wide. How many inches (x) should exist between each pair of letter?

Answers

Answer:

To evenly space the letters of "EAGLES" across a 63 inch wide banner with no margins, you would need to divide the remaining space between each letter of the word equally. With five letters total and each letter being 8 inches wide, the total width occupied by the letters is 40 inches. Therefore, the remaining space is 23 inches (63-40=23). To find out how many inches should exist between each pair of letter, you would divide the remaining space by the number of gaps between the letters, which is 4 (one less than the total number of letters in the word).

23 inches ÷ 4 gaps = 5.75 inches between each pair of letters.

Step-by-step explanation:

Using the above supply/demand graph, what is the price at the point of equilibrium

Answers

The price at the point of equilibrium is 105

What is the price at the point of equilibrium?

From the question, we have the following parameters that can be used in our computation:

The supply/demand graph

The price at the point of equilibrium is the price where the lines/curves of the supply and the demand functions intersect

Using the above and the graph as a guide, we have the following:

Price = 105 when demand = supply

Hence, the price at the point of equilibrium is 105

Read more abou demand and supply at

https://brainly.com/question/15247997

#SPJ1

Question

Using the above supply/demand graph, what is the price at the point of equilibrium? a. 105 b. 100 c. 95 d. 80

Other Questions
Of these choices, neuronal communication between the brain and the muscles of the leg is best conceptualized as _____. 1. the transcription of genes 2. chemical signaling 3. electrical and chemical signaling 4. the transcription and translation of genes 5. electrical signaling Mary bought 5 packages of 8 cupcakes. How many cupcakes did Mary buy in all?A] 40B] 48C] 35D] 32 the product-process matrix shows the relationship between process structures and product volume and variety characteristics. (True or False) Aluminum hydroxide is a base that is the active ingredient in some over-the-counter antacids. Suppose you have 22.0mL of 0.170M HCl solution in a flask and you add an antacid tablet to the HCl. After allowing the antacid to react with the HCi solution, you titrate the solution with 0.20 M NaOH. It requires 8.45 mL of NaOH to reach the end point. How many moles of HCI were neutralized by the antacid tablet? mol plsssss help me ........ List three examples of artificial pollutants and three examples of natural pollutants. What event would activate a G protein?SHOW HINTa) replacement of GDP with GTPb) hydrolysis of GDP to GTPc) phosphorylation of GDP to GTPd) hydrolysis of GTP to GDPe) phosphorylation of GTP to GD The Galapagos Islands are a series of islands near the coast of South America. The finches (a type of bird) on the Galapagos look similar to the finches in South America, but the birds on each island have some variations, including unique beak structures. Explain what differences on the islands would have led to this beak structure variation. apply 500 newtons of force until the speed reaches approximately 20 m/s. then, remove the force. describe the motion of the box ENGLISH - SPRING 2023 - ASSIGNMENT ON ARGUMENTATION (This is due tomorrow)First reread the chapter "Argumentation", then answer the following questions. Please make complete sentences and use formal language. Fragments, contractions, and colloquialisms are not allowed.I-What is the best way to build a strong argumentation: through persuading or convincing? Please justifyyour answer.II-What is the prerequisite for having a good debate ? Justify your answer.III-A. Explain the meaning of "syllogism". B. Give two instances of syllogism. Please choose your own instances.IV-For the sake of a coherent argumentation, it is necessary to create a topic that needs a thesis and an antithesis. Create your own topic with a thesis and an antithesis.V- Provide the definition of "logical fallacy". Give your two own examples of "logical fallacy". the secret service has compiled data for an accurate and useful profile of at-risk kids who become school shooters. T/F for a new customer top golf charges $10 for a player card and $2 per hour. The scale drawing below represents the side of a ramp. In the scale drawing, two inches equals five feet.What is the actual area, in square feet, of the side of the ramp? Jennie has 4 gallons of punch that she will pour into pint-size glasses. How many pints will she have? transference of emotions associated with a particular person, object, or situation; substituting a different target for impulses when the original would be dangerous or unacceptablewhat defense mechanism is this? Your store marks up couches 100% on cost. It pays $200 for each couch. How much will it sell the couches for? a. $300 Ob. $400 Oc. $600 Od $200 Anyone have songs that could fit any of these people. Voltaire John Locke Baron de Montesquieu Jean Jacques Rousseau (((ITS FOR A ENLIGHTENMENT PROJECT))) what is the optimal goal for kids with chronic health conditions? If a firm plans to issue new stock,flotation costs investment bankers' fees should not be ignored.There are two approaches to use to account for flotation costs.The first approach is to add the sum of flotation costs for the debt,preferred, and common stock and add them to the initial investment cost. Because the investment cost is increased,the project's expected rate of return is reduced so it may not meet the firm's hurdle rate for acceptance of the project.The second approach involves adjusting the cost of common equity as follows: Cost of equity from new stock=re= +g Po(1-F) The difference between the flotation-adjusted cost of equity and the cost of equity calculated without the flotation adjustment represents the flotation cost adjustment. Quantitative Problem: Barton Industries expects next year's annual dividend,D1,to be $1.60 and it expects dividends to grow at a constant rate g =4.3% The firm's current common stock price,Po,is $25.00.If it needs to issue new common stock,the firm will encounter a 4.6% flotation cost,F.What is the flotation cost adjustment that must be added to its cost of retained earnings? Do not round intermediate calculations.Round your answer to two decimal places 10.70 Write an essay about Ida b Wells.It can be an old essay or a new one.or even a copy & paste essay.